Tải bản đầy đủ (.pdf) (43 trang)

Một số ứng dụng hàm lồi để nghiên cứu martingale dưới và giải toán sơ cấp

Bạn đang xem bản rút gọn của tài liệu. Xem và tải ngay bản đầy đủ của tài liệu tại đây (942.81 KB, 43 trang )

4
LỜI NĨI ĐẦU
Trong lý thuyết tối ưu hố, giải tích lồi chiếm một vị trí rất quan trọng.
Đối tượng chủ yếu của giải tích lồi là tập hợp lồi và hàm lồi. Hàm lồi có rất
nhiều ứng dụng trong tốn học cao cấp và toán học sơ cấp. Một trong những
ứng dụng của nó đối với lý thuyết xác suất là Martingale dưới. Cịn đối với
tốn học sơ cấp các kết quả thu được trong việc nghiên cứu tập hợp lồi và
hàm lồi đươc áp dụng rộng rãi trong việc chứng minh bất đẳng thức sơ cấp và
giải các bài tốn cực trị có ràng buộc.
Xuất phát từ lý do đó chúng tơi đề xuất nghiên cứu đề tài "Một số ứng
dụng hàm lồi để nghiên cứu Martingale dưới và giải tốn sơ cấp".
Luận văn gồm có 2 chương
Chương 1 trình bày những kiến thức cơ bản về hàm lồi, một số bất đẳng
thức quan trọng của hàm lồi, đặc biệt là các dạng của bất đẳng thức Jensen và
ứng dụng hàm lồi để nghiên cứu Martingale dưới.
Chương 2 trình bày những ứng dụng của hàm lồi để giải toán sơ cấp như
chứng minh các bất đẳng thức và giải các bài tốn cực trị có ràng buộc.
Trong mỗi phần để có hệ thống chúng tơi đều nêu ra các kiến thức cơ bản
được trình bày ngắn gọn sau đó là các bài toán minh hoạ.
Luận văn này được thực hiện và hoàn thành tại trường Đại học Vinh dưới
sự hướng dẫn khoa học của TS. Trần Xuân Sinh. Tôi xin bày tỏ lời biết ơn sâu
sắc tới thầy về sự hướng dẫn tận tâm và nhiệt tình của thầy đối với tơi trong
suốt q trình học tập và nghiên cứu.
Nhân dịp này tôi cũng xin trân trọng gửi lời cảm ơn tới PGS.TS. Nguyễn
Văn Quảng, PGS.TS. Phan Đức Thành, TS. Nguyễn Trung Hồ, các thầy cơ
giáo trong tổ Xác suất Thống kê và Toán ứng dụng, Khoa toán, Khoa Sau đại
học, Trường Đại học Vinh và các bạn đã quan tâm, góp ý tạo điều kiện giúp
đỡ tơi hồn thành luận văn này.
Tôi xin chân thành cảm ơn.
Vinh, tháng 12 năm 2004
Tác giả




5

Chương 1
HÀM LỒI VÀ ỨNG DỤNG HÀM LỒI ĐỂ
NGHIÊN CỨU MARTINGALE DƯỚI
1.1. Hàm lồi
1.1.1. Tập hợp lồi
1.1.1.1. Đoạn thẳng trong Rn : Cho x(1), x(2)  Rn.
Tập hợp
x (1) x ( 2) = x  Rn : x = 1x(1) + 2x(2); 1, 2  0; 1 + 2 = 1

được gọi là đoạn thẳng nối hai điểm đã cho.
Chú ý rằng nếu ký hiệu 1 = ; 2 = 1 -  thì   [0, 1] và đoạn thẳng nối
x(1), x(2) sẽ là
x (1) x ( 2) = x  Rn : x = x(1) + (1-)x(2);   [0, 1].

1.1.1.2. Tập hợp lồi
+ Định nghĩa: Tập hợp M  Rn được gọi là tập lồi nếu đoạn thẳng nối hai
điểm bất kỳ thuộc M nằm trọn trong M, nghĩa là với
x(1), x(2)  M : x = x(1) + (1-)x(2),   [0, 1] thì x  M.
Ví dụ: Cho A = (aij); x = (xj); B = (bi) là các ma trận cấp mn; n1; m1
tương ứng. Khi đó tập
M = X  Rn; AX = B, X  0
là tập hợp lồi.
1.1.2. Hàm lồi
1.1.2.1. Định nghĩa
+ Định nghĩa: Cho tập lồi M  Rn và hàm số f xác định trên M. Hàm f
được gọi là hàm lồi xác định trên tập lồi M nếu với mọi x, y  M và   [0, 1]

ta có
f(x + (1-)y))  f(x) + (1-)f(y).

(2.1)


6
+ Ví dụ:
Hàm số f(x) = x2 là hàm lồi.
Thật vậy
Lấy x1, x2  R1; x1  x2; 1, 2  0 và 1 + 2 = 1, ta phải chứng minh
f(1x1 + 2x2)  1f(x1) + 2f(x2)
Thật vậy, ta có
f(1x1 + 2x2)  1f(x1) + 2f(x2)
 (1 x1  2 x2 ) 2  1 x12  2 x22
 12 x12  22 x22  212 x1 x2  1 x12  2 x22
 (1  12 ) x12  (2  22 ) x22  212 x1 x2  0
 1 (1  1 ) x12  2 (1  2 ) x22  21 (1  1 ) x1 x2  0
 1 (1  1 )( x1  x2 ) 2  0.

Vì 1  0, 1 - 1  0, x1  x2 do đó bất đẳng thức nêu trên hiển nhiên đúng.
Vậy f(x) = x2 là hàm lồi.
1.1.2.2. Định lý (Định lý điểm giữa). Hàm f liên tục trên tập lồi M là lồi
khi và chỉ khi
x y 1
  (f(x) + f(y)).
 2  2

f


(2.2)

Chứng minh. Điều kiện cần là rõ ràng vì từ bất đẳng thức lồi đúng với mọi

 thuộc [0, 1].
Điều kiện đủ. Ta cần chứng minh nếu có (2.2) thì hàm liên tục f là lồi.
Từ (2.2) ta có
 x (1)  x ( 2)  x (3)  x ( 4)
4


f 

 1
  [f(x(1)) + f(x(2)) + f(x(3)) + f(x(4))].
 4

Bằng quy nạp hoàn toàn với m = 2k, k là số tự nhiên nào đó, ta nhận được
bất đẳng thức


7
 1 m (i )  1 m
f   x    f(x(i))
 m i 1
 m i 1

(2.2a)

Ta chứng minh quy nạp bất đẳng thức (2.2a) đúng với mọi số tự nhiên m.

Rõ ràng với m đủ lớn thì (2.2a) đúng. Giả sử với m = N bất đẳng thức
đúng, tức là
1
N

f 

 1
x (i )  

i 1
 N
N

N



f(x(i)),

i 1

ta chứng minh bất đẳng thức đúng với m = N - 1. Đặt
1
(x(1) + x(2) +... + x(N-1)),
N 1

x(N) =
ta có


x(N) =

1 (1)
(x + x(2) +... + x(N)),
N


1
N

f(x(N)) = f 

N

x

(i )

i 1

 1

 N

N



f(x(i)) =


i 1

1
N

N 1



f(x(i)) +

i 1

1
f(x(N)).
N

Từ đó ta được
1 N 1
 1 N 1 (i ) 
x

f(x(i)).



 N  1 i 1
 N  1 i 1

f 


Như vậy (2.2a) đúng với mọi m số tự nhiên.
Để chứng minh f lồi, ta phải chứng minh f thỏa mãn bất đẳng thức lồi
(2.1). Lấy x, y thuộc M và k, m là các số tự nhiên, k < m. Từ (2.2a) ta có
kx  (m  k ) y 
mk  1
k
f 
[k f(x) + (m-k) f(y)]
y 
 = f  x


Đặt  =

m



m

m



m

k
, ta có  là số hữu tỉ thỏa mãn 0 <  < 1 và
m


f(x + (1-)y)   f(x) + (1-) f(y).
Vì f là hàm liên tục, do vậy bất đẳng thức lồi trên cũng đúng với mọi giá
trị thực  thuộc đoạn [0, 1]. 


8
Hệ quả. Cho M là tập hợp lồi, f(x) là hàm liên tục, lồi và xác định không
âm trên M, khi đó f 2(x) cũng là hàm lồi trên M.
Thật vậy, do M lồi nên x, y thuộc M thì

x y
thuộc M. Lại do f lồi nên
2

theo định lý 1.1.2.2 thì
x y 1
  [f(x) + f(y)].
 2  2

f 
Do f là hàm không âm nên

x y 1
2
  [f(x) + f(y)] .
 2  4

f 2
Mặt khác ta có


1
1
1
1
[f(x) + f(y)]2 - f 2(x) - f 2(y) = - [ f 2(x) + f 2(y) - 2f(x) f(y)] =
4
2
2
4

=-

1
[f(x) - f(y)]2  0.
4

Vậy
1 2
x y 1
2
2
  [f(x) + f(y)]  [ f (x) + f (y)],
2
 2  4

f 2

nghĩa là hàm f 2(x) là hàm lồi trên M. 
1.1.2.3. Định lý. Cho fi(x), (i = 1, 2,..., k) là những hàm lồi trên tập lồi M.

Khi đó
k

f(x) =



ifi(x), i  0, i = 1, 2,..., k.

i 1

là hàm lồi.
Chứng minh. Từ định nghĩa 1.1.2.1 suy ra kết quả của định lý. 
Đặc biệt chúng ta có: Tổng hữu hạn các hàm lồi xác định trên tập lồi M là
hàm lồi.
1.1.2.4. Định lý. Cho f(x) là hàm lồi trên tập lồi M và số thực  cố định.
Khi đó tập M = x  Rn: f(x)    là tập lồi.


9
Chứng minh. Lấy x, y  M, khi đó do f là hàm lồi nên với   [0, 1] ta có
f(x + (1-)y)   f(x) + (1-) f(y)   + (1-) = .
Từ đó suy ra x + (1-)y  M.



Hệ quả. Cho fi, i = 1, 2,..., k là các hàm lồi xác định trên tập lồi M. Khi đó
tập M = x  M: fi(x)  i, i = 1, 2,..., k, i R là tập lồi.
1.1.2.5. Định lý. Cho hàm số f(x) xác định và liên tục, có đạo hàm cấp 2
trên (a, b). Khi đó f(x) lồi trên (a, b) nếu f ''(x)  0, x  (a, b).

Chứng minh. Lấy x, z  (a, b), (x < z). Ký hiệu c =

xz
. Ta phải chứng
2

minh
f(c) 

1
(f(x) + f(z)).
2

Xét đoạn thẳng [x, c]. Theo định lý Lagrăng, tồn tại 1  [x, c] sao cho
f(c) - f(x) = f '(1).(c - x).
Từ đó suy ra
f '(1) =

f (c )  f ( x )
f (c )  f ( x )
f (c )  f ( x )
=
=
xz
1
cx
x
( z  x)
2
2


(1a)

Lại xét đoạn [c, z]. Cũng theo định lý Lagrăng tồn tại 2  [c, z] sao cho
f '(2) =

f ( z )  f (c )
f ( z )  f (c )
f ( z )  f (c )
=
=
xz
1
z c
z
( z  x)
2
2

(1b)

Theo giả thiết f ''(x)  0, với mọi x thuộc (a, b), cho nên f '(x) là hàm đồng
biến trên [a, b], ta suy ra với 1 < 2 thì
f '(1) < f '(2)

(1c)

Từ (1a), (1b), (1c) suy ra
f (c )  f ( x )
f ( z )  f (c )

<
1
1
( z  x)
( z  x)
2
2

Do z - x > 0, nên từ (1d) ta được

(1d)


10
1
1
( f(c) - f(x)) < ( f(z) - f(c)),
2
2

tức là
f(c) <

1
( f(z) + f(x)).
2



Đó là điều phải chứng minh.


Ví dụ. Cho hàm mật độ của đại lượng ngẫu nhiên có phân phối chuẩn
1 
e
2

f(x) =

( x )2
2 2

,

trong đó  là kỳ vọng, 2 là phương sai. Khi đó f(x) là hàm lồi với mọi x thỏa
mãn x   -  hoặc x   + .
Thật vậy, để chỉ ra f(x) là hàm lồi với x đã cho, ta chỉ cần chứng minh
F(x) = e



( x )2
2 2

là hàm lồi với x đã cho.
Ta có
F'(x) = - e
F''(x) = - e




( x )2
2

2



=

.

1



( x  )2

e

2 2

2

2



( x )2

-


2 2

.

x



2

x

2

(- e



,
( x )2
2 2

.

x

2

)


 (x   )2


 .

1
2




Rõ ràng F(x)  0 khi và chỉ khi
(x   )2



2

 1  (x - )2  2  x   -  hoặc x   +  (vì  > 0).

Hệ quả. Các hàm số sau đây là hàm lồi
+ y = xk với x > 0; k = 1, 2, ...
+ y = ax với a > 1
+ y = - loga với a > 1, x > 0


11
+ y = sinx ; x  (, 2).
1.1.2.6. Định lý. Cho f khả vi trên tập lồi M, để f là hàm lồi điều kiện cần

và đủ là
f(x) , y - x   f(y) - f(x), x, y  M,
(2.3)
,
,
,
trong đó f(x) = f’(x) = ( f x1 , f x2 ,..., f xn ).
Chứng minh định lý 1.1.2.6 có thể xem [9]. 
Cho f(x) là hàm lồi xác định trên tập lồi M, lấy x, y cố định thuộc M. Xét
z = x + (1-)y, 0    1 thì z  M. Ký hiệu () = f(x + (1-)y). Khi đó
() là hàm một biến
1.1.2.7. Định lý. Nếu f(x) là hàm lồi xác định trên tập lồi M thì () là
hàm lồi xác định trên đoạn [0, 1].
Việc chứng minh định lý 1.1.2.7, suy ra trực tiếp từ chứng minh điều kiện
đủ của định lý 1.1.2.6. 
1.1.2.8. Định lý. Cho M lồi thuộc R, hàm lồi f : M  R. Khi đó ta có
f ( x)  f (a) f (b)  f (a) f (b)  f ( x)
,


xa
ba
bx

trong đó a, b, x  M, a < x < b.
Việc chứng minh định lý 1.1.2.8 có thể xem trong [10].
Hệ quả [10]. Cho M lồi thuộc R; a, b là các điểm trong của M; u, s, t 
(a, b), s < t < u; hàm lồi f : M  R. Khi đó ta có
u t
ts

f(s) +
f(u),
us
us

(2.4)

f (t )  f ( s) f (u )  f (t )

.
ts
u t

(2.5)

f(t) 
hay tương đương là

1.1.2.9. Định lý. Cực tiểu địa phương của hàm lồi f trùng với cực tiểu
tuyệt đối trên tập lồi M.
Chứng minh. Giả sử x(0)  M là điểm cực tiểu địa phương của f trên M.
Khi đó tồn tại lân cận  x( 0 ) sao cho
f(x(0))  f(x); x  M   x( 0 )


12
Lấy bất kỳ x  M. Xét y = x + (1 - )x(0); 0    1. Rõ ràng với  đủ bé
ta có x rơi vào lân cận của x(0). Mặt khác do M lồi nên y  M.
Từ đó suy ra y  M   x( 0 ) . Do vậy f(x(0))  f(y).
Nhưng f là hàm lồi nên

f(x(0))  f(y)  f(x) + (1 - )f(x(0))
f(x(0))  f(x), x  M.

Vậy

Điểm x(0)  M lồi được gọi là điểm trong của M nếu với mỗi x  M thì tồn
tại y  M mà x(0) = x + (1 - )y; 0 <  < 1.
1.1.2.10. Định lý. Nếu hàm lồi f đạt cực đại tại điểm trong x(0) của tập lồi
M thì f không đổi trên tập M.
Chứng minh. Giả sử x(0) là điểm trong của M mà f đạt cực đại tức là
f(x), f(y)  f(x(0)), với x, y  M.
Vì x(0) là điểm trong nên với mỗi x  M thì tồn tại y  M mà
x(0) = x + (1-)y, 0 <  < 1.
Lại vì f lồi nên
f(x(0))  f(x) + (1-)f(y).
Theo giả thiết ta có
f(x(0))  f(x) + (1-)f(y)  f(x) + (1-) f(x(0)).
Vì 0 <  < 1 nên ta có được f(x(0))  f(x), x  M.
Từ đó suy ra f(x(0)) = f(x), với x  M, tức là f không đổi trên M
Hệ quả. Cho hàm lồi f xác định trên tập hợp lồi M, khi đó cực đại của f
(nếu có) sẽ đạt tại điểm cực biên của M.
1.2. Một số bất đẳng thức quan trọng của hàm lồi
1.2.1. Bất đẳng thức Jensen
1.2.1.1. Định lý. f(x) là hàm lồi trên tập lồi M khi và chỉ khi
k

f(  ix(i)) 
i 1

k



i 1

i f(x(i)), x(i)  M,

(2.6)


13
trong đó i  0,

k



i = 1.

i 1

Chứng minh. Giả sử có bất đẳng thức (2.6), khi đó với k = 2, theo định
nghĩa thì f là hàm lồi.
Ngược lại, giả sử f(x) là lồi trên M. Xét
k

x=



i x , i  0,

(i)

i 1

k



i = 1.

i 1

Rõ ràng do M lồi nên x  M.
Để chứng minh (2.6), ta quy nạp theo k.
Với k = 2, bất đẳng thức đúng.
Giả sử bất đẳng thức đúng với k-1, ta chứng minh đúng với k.
Ta có
k



i f(x(i)) =

i 1

k 1



i f(x(i)) + k f(x(k)).


i 1

Không mất tổng quát, giả sử 0 < k < 1. Khi đó
k



k 1

i f(x ) = (1-k) 
(i)

i 1

i 1

i
f(x(i)) + k f(x(k))
1  k
k 1

= k f(x(k)) + (1-k)  i f(x(i)),
i 1

với i =

i
 0, i = 1, 2,..., k-1, rõ ràng
1  k


k 1



i = 1. Do vậy, theo giả thiết

i 1

quy nạp ta được
k



i f(x(i))  (1-k)

i 1

k 1


i 1

 f[k x(k) + (1-k)

k 1


i 1


Định lý được chứng minh.

i f(x(i)) + k f(x(k))



k

i x(i)]  f(  i x(i)).
i 1


14
Chú ý: Các hệ quả sau đây là trường hợp đặc biệt của bất đẳng thức
Jensen.
Hệ quả
1) Nếu hàm số y = f(x) lồi trên tập lồi M thì với mọi x1, x2,..., xn  M ta có
bất đẳng thức
f ( x1 )  f ( x2 )  ...  f ( xn )
 x  x 2  ...  x n 
, (n  2). (2.6a)
f 1
 
n
n



2) Nếu hàm số y = f(x) lồi trên M lồi thì với x1, x2,..., xn  M ta có bất
đẳng thức sau

 m x  m2 x2  ...  mn xn  m1 f ( x1 )  m2 f ( x2 )  ...  mn f ( xn )
 
. (2.6b)
f  1 1
m1  m2  ...  mn
 m1  m2  ...  mn


Thật vậy, áp dụng bất đẳng thức Jensen (2.6) lấy với

1 = 2 =... = n =

1
,
n

ta có bất đẳng thức (2.6a). Lấy với
mi
, mi > 0, i = 1, 2,..., n,
m1  m2  ...  mn

i =

ta có bất đẳng thức (2.6b)
1.2.2. Bất đẳng thức Petrovica
1.2.2.1. Định lý. Cho f(x) là hàm lồi một biến trên [0, a]. Giả sử xi  [0, a],
với i = 1, .., n, sao cho

i1 xi  [0, a]. Khi đó i1
n


n

f(0).
Chứng minh. Do

i1 xi  [0, a] và 0  [0, a] nên
n

xi

i1
n

Theo định nghĩa hàm lồi ta có

xi



i j
n
i1

xj
xi

 1.

f(xi)  f( i 1 xi ) + (n-1)

n


15

 x
f(xi) = f  n i
  xi
 i 1



xi 
.0 

i 1
 xi 
i 1

 xj

n

j i
n

xi
n

 xi


x
 n  j i j
f   xi   n
f 0
 i 1   xi

(*)

i 1

i 1

Cộng vế với vế của n bất đẳng thức dạng (*) ta được
n


i 1


n   xj
 n 
j i
f  xi    xi n
f   xi   f (0).   n
i 1
i 1   x
 xi  i 1 

i 1

 i 1 i
n

xi







Từ đó suy ra
n


i 1

 n 
f xi   f   xi   n  1 f 0 .
 i 1 



Chú ý: Ta cũng có một suy rộng của bất đẳng thức Petrovica như sau:
Cho f(x) là hàm lồi trên [0, a], với a > 0 và P1, P2,..., Pn là các số không
âm sao cho
 n

xi  0,  pi xi  , i = 1, n và
 i 1



n

 p i xi  a .
i 1

Khi đó ta có
 n
  n


p
f
(
x
)

f
p
x
i
i

  i i     pi  1 f (0) .
i 1
 i 1
  i1

n


1.2.3. Bất đẳng thức Vasic
1.2.3.1. Định lý. Cho f(x) là hàm lồi một biến và x1, x2,..., xn; p1, p2,..., pn
là hai dãy số không âm thoả mãn các điều kiện sau:
i) xi  [0, a), với i = 1, 2, ..., n
ii) pi  1, với i = 1, 2, ..., n
n1 1

iii)

 p i xi



i 1

iv)

n2 1



i  n1

pi xi  .... 

1 n
 pi xi  (0, a) .
k i 1


Khi đó ta có

n



i  nk 1

p i xi


16
n
1 n
 

p
f
(
x
)

kf
p
x

k

Pi  f (0) .




 i i


i i
i 1
i 1
 k i 1
 

n

Chứng minh. Theo bất đẳng thức Petrovica dạng tổng quát ta có
nv 1 1



i  nv

 nv 1 1
  nv 1 1 

pi f xi   f   pi xi   1   pi . f (0)
i  nv
 i nv
 


(2.7)


với v = 1, 2, ..., k.
Theo giả thiết ta có
nv 1 1



i  nv

1 n
p i xi   p i xi .
k i 1

Cộng từng vế k bất đẳng thức dạng (2.7) ta có điều cần chứng minh
n
1 n
 

p
f
(
x
)

kf
p
x

k


pi  f (0) . 



 i i


i i
i 1
i 1
 k i 1
 

n

1.2.4. Bất đẳng thức Karamata
1.2.4.1. Định lý. Giả sử f(x) là hàm số liên tục, có đạo hàm cấp hai f ''(x)  0,
với x  R. Giả sử x , y , z là các số thực thoả mãn điều kiện sau:






x  a1
x + y  a1 + a2
x + y + z = a1 + a2 + a3.

ở đây a1  a2  a3. Khi đó ta có
f( x ) + f( y ) + f( z )  f(a1) + f(a2) + f(a3).


(2.8)

Dấu "=" trong bất đẳng thức xảy ra khi và chỉ khi x = a1, y = a2, z = a3.
Chứng minh. Trước hết ta chứng minh rằng
f ( x)  f (a1 )  ( x  a1 ) f ' (a1 ) , với x  R

Thật vậy, do f ''(x) > 0, với x  R, nên f '(x) là hàm đồng biến.
Nếu x > a1 theo định lý Lagrăng tồn tại   (a1, x) sao cho
f ( x)  f (a1 )  ( x  a1 ) f ' ( )

(1a)


17
Vì   (a1, x) nên  > a1, vậy nên f '()  f '(a1).
Do x > a1 nên x - a1 > 0, từ đó ta có
( x  a1 ) f ' ( )  ( x  a1 ) f ' (a1 )

Từ đó ta suy ra (1a) đúng
Nếu x < a1 theo định lý Lagrăng tồn tại   (x, a1) sao cho
f (a1 )  f ( x)  (a1  x) f ' ( ) , hay f ( x)  f (a1 )  ( x  a1 ) f ' ( )

Vì  < a1 nên f '()  f '(a1), đồng thời x - a1 < 0, do đó
f '()(x - a1)  f '(a1)(x - a1).
Từ đó suy ra
f(x) - f(a1)  f '()(x - a1)  f '(a1)(x - a1).
Vậy (1a) đúng.
Tương tự (1a) ta có
f(y)  f(a2) + (y - a2) f '(a2), với y R


(1b)

f(z)  f(a3) + (z - a3) f '(a3), với z R

(1c)

Bây giờ ta trở lại chứng minh định lý 1.2.4.1.
Gia sử x , y , z là các số thực thoả mãn điều kiện






x  a1
x + y  a1 + a2
x + y + z = a1 + a2 + a3.

với a1  a2  a3.
Theo (1a), (1b), (1c) ta có
f( x ) + f( y ) + f( z ) 
f(a1) + f(a2) + f(a3) + ( x - a1) f '(a1) + ( y - a2) f '(a2) + ( z - a3) f '(a3) (2a)
Bằng phương pháp thêm bớt ta có
Vế phải (1d) = f(a1) + f(a2) + f(a3) + f '(a3)( x + y + z - a1 - a2 - a3)
+ [f''(a2) - f '(a3)] ( x + y - a1 - a2) + [f''(a1) - f '(a2)] ( x - a1)
Do f '(x) là hàm đồng biến trên toàn trục số nên từ a1 > a2 > a3 có

(2b)



18
f '(a1)  f '(a2)  f '(a3).
Từ đó suy ra
Vế phải (2b)  f(a1) + f(a2) + f(a3)

(2c)

Từ (2a), (2c) ta có
f( x ) + f( y ) + f( z )  f(a1) + f(a2) + f(a3).
Rõ ràng từ lập luận trên ta suy ra dấu "=" trong (2.8) xảy ra khi và chỉ khi
x = a1, y = a2, z = a3. Bất đẳng thức được chứng minh.

1.3. Ứng dụng hàm lồi để nghiên cứu Martingle dưới
1.3.1. Martingale
1.3.1.1. Định nghĩa. Cho không gian xác suất (, F, P) trên đó tách ra họ
(F n) các  - đại số F n, n > 0 sao cho F 0  F 1  ...  F.
Giả sử X0, X1, ... là dãy các đại lượng ngẫu nhiên (ĐLNN) cho trên (, F, P).
Ta nói rằng, q trình ngẫu nhiên X = Xn, F n, n  N là dãy tương thích, nếu
Xn là F n - đo được với mọi n  N và ký hiệu là Xn  F n.
Quá trình ngẫu nhiên: X = Xn, F n, n  N gọi là dãy dự báo nếu Xn  F n-1
với mỗi n  N (tức là Xn đo được F n-1).
Dãy dự báo được gọi là dãy dự báo tăng nếu X0 = 0 và X1  Xn+1 (P-h.c.c).
1.3.1.2. Định nghĩa. X = Xn, F n, n  N được gọi là Martingale (đối với
 F n, n  N) nếu
(i) Xn, F n, n  N là dãy tương thích
(ii) EXn < , n  N
(iii) Với m  n, m, n  N thì E(Xn F m) = Xm, P- h.c.c.
và ta gọi X là Martingale dưới (đối với F n, n  N) nếu các điều kiện (i), (ii)
được thực hiện và

(iii') m  n; m, n  N thì E(Xn  F m)  Xm , P- h.c.c.
1.3.2. Tính chất cơ bản


19
1.3.2.1. Tính chất 1. Nếu X = Xn, F n , n  N là martingale, thì hàm trung
bình EXn khơng phụ thuộc n  N.
1.3.2.2. Tính chất 2. Nếu X = Xn, F n, n  N là martingale dưới thì hàm
trung bình EXn khơng giảm theo n  N.
1.3.2.3. Tính chất 3. Nếu X = Xn, Fn, n  N là martingale thì hàm EXnp,
1  p <  không giảm theo n  N.
1.3.3. Mối liên hệ giữa martingale - hàm lồi - martingale dưới
1.3.3.1.Định nghĩa. Ta hãy nhắc lại Định nghĩa 1.1.2.1, Định lý 1.1.2.8 và
Hệ quả từ mục 1.1 về hàm lồi: Hàm f thực hữu hạn trên khoảng J  (-,)
được gọi là lồi trên J nếu bất kỳ x1, x2  J và   [0, 1] ta có
f(x1 + (1- )x2)  f(x1) + (1 - )f(x2).

(2.9)

Khi đó theo Hệ quả của Định lý 1.1.2.8, ta có
f(t) 

ts
u t
f(u) +
f(s), s < t < u
us
us

(2.10)


hay tương đương
f (t )  f ( s)
f (u )  f (t )

.
ts
u t

(2.11)

trong đó
t=u

u t
ts
+s
, s < t < u.
us
us

Nếu f là hàm lồi, liên tục trên khoảng mở (a, b), từ (2.10) sẽ có
lim f(t)  f(s); lim f(s)  f(t)
t s

st

lim f(u)  f(t); lim f(t)  f(u)

ut


t u

Hơn nữa từ (2.11) với f là hàm lồi trên (a, b) ta sẽ có đạo hàm f ' là khơng
giảm trên (a, b). Ngồi ra, người ta cũng đã chứng minh được rằng:
(i) Nếu f là lồi trên khoảng mở (a, b) thì nó sẽ có đạo hàm trái ft' và đạo
hàm phải fp' tại mọi điểm trên (a, b) với ft'  fp' và mỗi đạo hàm là không giảm.


20
(ii) Nếu f là lồi trên khoảng (a, b), mọi   (a, b) ta có
f(t)  f() + (t - )fp'(), t  (a, b).

(2.12)

1.3.3.2. Định lý. Nếu X là biến ngẫu nhiên khả tích, c là hằng số và f là
hàm lồi trên (-, ) thì ta có
-

-

Ef (X - EX + c) <  với f = max(- f, 0)

(2.13)
-

Ngoài ra, nếu a(t) và t - a(t) là hàm không giảm trên (-, ), với (E  (X)
-

<  thì E(X) < ; Ef (a(X) - Ea(X) + c) <  và

Ef(X - EX + c)  Ef(a(X) - EaX + c).

(2.14)

Chứng minh. Theo (2.12) thì f(t)  f(0) + tfp’(0), với t  (- , ), do đó
có (2.13). Do t  0, t - (t)  - a(0) có Ea+ (X +)  E(X +) + a( 0) < .
Kết hợp với giả thiết ta suy ra E(X) < . Từ (2.12) lại có được
-

Ef ((X) - E(X) + c) < .
Đặt

(t) = t - (t) - EX + E(X), t  (- , ).
Khi đó E(X) <  và E(X) = 0. Nếu P(X) = 0 = 1 thì (2.14) là tầm
thường.
Ngược lại, sẽ có t1, t2,  (- , ) mà (t1) < 0, (t2) > 0.
Đặt có t0 = inft : (t) > 0, do tính đơn điệu của t - (t) thì t1  t0  t2 và
nếu (t) > 0 thì t  t0, nếu (t) < 0 thì t  t0.

(2.15)

Lại theo (2.12)
f(X - EX + c)  f((X) - E(X) + c) + (X)fp’((X) - E (X) + c)

(2.16)

Theo (2.15) thì X  t0 khi (X) > 0 và X  t0 với (X) < 0. Vì cả hai fp’ và 
là khơng giảm nên ta có

(X)fp’((X) - E(X) + c)  (X)fp’((t0) - E(X) + c)

Từ (2.16), (2.17) và kết luận (2.14) chỉ ra rằng E(X) = 0.

(2.17)


Hệ qủa 1. Nếu f là hàm lồi trong (-, ) đối với mỗi biến ngẫu nhiên X
khả tích và mọi hằng số c ta có


21
Ef(X - EX + c)  f(c).

(2.18)

Đặc biệt
Ef(X)  f(EX) (Bất đẳng thức Jensen).

(2.19)

Chú ý rằng bất đẳng thức (2.19) cũng đúng với kỳ vọng có điều kiện, tức

E(f(X)F ) > f(E(X F )).
Việc chứng minh Hệ quả 1 có thể xem trong [7, tr.127]
Hệ quả 2. Nếu X là biến ngẫu nhiên khả tích thì với 1  p < , ta có
EX - EXp  EY - EXp.

(2.20)

trong đó Y = XI[a < x < b] + aI[x < a] + bI[x > b], -   a < b  .
Chứng minh. Lấy c = 0,  = maxa, min(t, b), f(t) = tp, p  1 thay vào

(2.14) ta có kết quả cần chứng minh.



Ta viết gọn (EX)p bởi EpX, khi đó với 1  p <  bất đẳng thức Jensen
(2.19) mang lại
EXp  EpX, hoặc E1/pXp  EX

(2.21)

Thay p và X trong (2.21) bởi p'/p và Xp ta có
E1/pXp  E1/pXp', 0 < p < p' < 

(2.22)

Đồng thời
Xp = E1/pXp, p > 0,

(2.23)

Từ (2.22) và (2.23) ta có Xp  Xp' , đối với 0 < p < p'. Ngoài ra Xp
thỏa mãn bất đẳng thức tam giác đối với p  1.
1.3.3.3. Định lý (Bất đẳng thức Hửlder). Nếu X, Y là các hàm đo được trên
không gian xác suất (, F, P), vậy thì với p > 1, p' > 1 mà (1/p) + (1/p') = 1, ta

EXY  Xp .Yp' .
Chứng minh. Chúng ta giả thiết rằng 0 < Xp .Yp' < . Đặt

(2.24)



22

U=

Y
X
,V=
,
Y p'
X p

(*)

với Up = 1 = Vp' .
Ta có hàm - lnx là hàm lồi trên (0, ), từ đó theo bất đẳng thức lồi (2.11)
thì với mọi a, b > 0 ta được
 a p b p' 
1
1
  - lnap - lnbp' = - lnab,
- ln 

p'
p
p' 
 p

hoặc tương đương
a p b p'


ab 
, 0  a, b  .
p
p'

Từ đó suy ra
EUV 

1
1
1
1
EUp + EVp' =
+
= 1.
p'
p'
p
p

Thay U, V trong (*) vào bất đẳng thức nêu trên ta có điều phải chứng minh
(2.24). 
1.3.4. Một số ví dụ áp dụng
Các ví dụ sau đây cho chúng ta thấy mối liên hệ giữa hàm lồi và
martingale dưới
1.3.4.1. Nếu X = Xn , F n , n  N là martingale và f là hàm lồi với
E f(Xn)  < , n  N,
thì dãy f(Xn) , F n, n  N là matingale dưới.
Đặc biệt dãy Xna, F n, n  N, a  1; Xnlog a Xn, Fn, n  N trong đó

log a = max0, loga, lập thành martingale dưới.
Thật vậy, vì Xn, Fn , n  N là martingale nên với m  n ta có
E(Xn F m) = Xm, suy ra f(E(Xn Fm)) = f(Xm).
Mặt khác, theo bất đẳng thức Jensen với m  n ta có
f(E(XnFm))  E(f(Xn)Fm).


23
Do đó ta có
Ef(Xn Fm)  f(Xm).
Vậy f(Xn), Fn, n  N là margingale dưới.
Tương tự do Xa , 1  a   là hàm lồi nên Xna, Fn, n  N là martingale
dưới.
1.3.4.2. Nếu X = Xn, Fn, n  N là martingale dưới và f là hàm lồi không
giảm với E f(Xn) < , n  N thì dãy f(Xn), Fn, n  N là martingale dưới.
Thật vậy, vì Xn, Fn, n  N là martingale dưới nên với m  n, ta có
E(Xn Fm)  Xm .
Vì f là hàm lồi khơng giảm nên
f(E(Xn Fm))  f(Xm).
Mặt khác, theo bất đẳng thức Jensen với m  n ta có
f(E(Xn Fm))  E(f(Xn) Fm).
Do đó
E(f(Xn Fm)  f(Xm).
Vậy f(Xn), Fn, n  N là martingale dưới.


24
Chương 2
ỨNG DỤNG HÀM LỒI ĐỂ GIẢI TOÁN SƠ CẤP
2.1. Các bài toán về chứng minh bất đẳng thức trong đại số sơ cấp

2.1.1. Các bất đẳng thức cổ điển
2.1.1.1. Bất đẳng thức Côsi
x1  x2  ...  xn

n

n

x1 x2 ...xn , (xi > 0, i = 1, 2, ..., n).

Chứng minh. Ta có hàm f(x) = - lnx là hàm lồi.
Áp dụng bất đẳng thức Jensen dạng (2.6a) ta có
 x  ...  xn  f ( x1 )  ...  f ( xn )
f 1

n
n



 x  x2  ...  xn   ln x1  ln x2  ...  ln xn
  ln  1

n
n


1
 x1  x2  ..  xn 
 ln 

  ln x1 x2 ...xn n
n





x1  x2  ...  xn n
 x1 x2 ...xn .
n

Dấu "=" xảy ra khi và chỉ khi x1 = x2 = ... = xn.



2.1.1.2. Bất đẳng thức Bunhiacôpxki
(a1b1 + a2b2+...+anbn)2  (a12 + a22 +...+ an2) (b12 + b22 +...+ bn2),
trong đó a1, a2, ..., an; b1, b2,..., bn là 2n số tuỳ ý.
Chứng minh. Ta có hàm f(x) = x2, với x  R là hàm lồi.
Áp dụng bất đẳng thức Jensen dạng (2.6b) ta có
2

2
2
 m1 x1  ...  mn xn 
m1 x1  ...  mn xn

 
m1  ...  mn
 m1  m2  ...  mn 


 (m1x1 +...+ mnxn)2 < (m1 +... + mn)(m1x12 + ...+ mnxn2)
Đặt mi = bi2 và xi =

ai
ta có
bi


25
 2 a1
2 a 
 b1
 ...  bn n   (b12 +...+ bn2)
bn 
 b1

2
 2 a12
2 an 
 b1

 .... bn
2
2 

b1
bn 



 a1b1 +...+ anbn  (a12 +....+ an2)(b12 + ...+ bn2).
Dấu “=” xẩy ra khi và chỉ khi

a
a1
 ...  n . 
b1
bn

2.1.1.3. Bất đẳng thức Mincôpxki
n

a1a2 ...an  n b1b2 ...bn  n a1  b1 a2  b2 ...an  bn  ,

trong đó a1, a2..., an; b1, b1,..., bn là 2n số dương.
Chứng minh. Xét hàm số f(x) = ln(1 + ex) khi đó
f '(x) =

ex
ex
;
f
'
'
(
x
)

1 ex
1 ex






2

 0,

với x  R nên f(x) là hàm lồi trên R.
Áp dụng bất đẳng thức Jensen dạng (2.6a)
 x  ...  x n  f ( x1 )  ....  f ( x n )
f 1
.

n
n



Chọn xi = ln

bi
; i = 1, 2, ..., n ta có
ai

b
b

ln 1 ...  ln n

a
a

1
n
n

ln 1  e






 ln 1  e





n

b
b


ln n 
ln 1 
a
a

 ln 1  e 1   ...  ln 1  e n 










n




 b b
ln n 1... n
 a1 ... an

  b1 
 b 
 ln 1    ...  ln 1  n  
 a 
  a1 
n 


 
n

 

 









a1...an  n b1...bn
n

a1a2 ...an

n

a1  b1 ...an  bn 
a1a2 ...an


26


n

a1 ...an  n b1 ...bn  n a1  b1 ...an  bn  .


Đó là điều cần chứng minh. 
2.1.1.4. Bất đẳng thức Hônde (Hửlder)
Cho các số thực dương ai, bi (i = 1, 2, ..., n) và p, q thỏa mãn

1 1
  1.
p q

Khi đó
a1b1 + a2b2 +...+ anbn  (a1p + a2p + ....+ anp)1/p.(b1q + b2q + ....+ bnq)1/q.
Chứng minh. Xét hàm lồi f(x) = xp , (x > 0; p > 1).
Áp dụng bất đẳng thức Jensen dạng (2.6b) ta có
p

p
p
 m1 x1  ..  mn xn 
m x  ..  mn xn

  1 1
.
m1  ...  mn
 m1  ...  mn 

1
p

 m x  ....  mn xn
m1 x1  ...  mn xn


  1 1

m1  ...  mn
m

...

m
1
n


p



p

 m1x1 +...+ mnxn  (m1x1p + ...+ mnxnp)1/p (m1 +...+ mn)(p-1)/p.
Do

1 1
p 1 1
  1 nên
 , thay vào bất đẳng thức trên ta được
p q
p
q
1


1

n
p p
n
q
 mi xi    mi xi    mi  .
i 1
 i 1
  i 1 
n

Đặt mi = biq ; xi = aibi1- q ta có
n

 bi ai bi
q

i 1

1q





n q
1q p 
   bi ai bi


i 1


1/ p

 n p pq pq 
  ai bi    ai b

i 1
 i1

n

Lại do

1/ p

1/ q

 n q
  bi 
 i 1 

1/ q

 n q
  bi 
 i 1 

1 1

  1 nên p + q - pq = 0, suy ra bp+q-pq = b0 = 1.
p q

Vậy
a1b1 + a2b2 +...+ anbn  (a1p + a2p + ....+ anp)1/p)(b1q + b2q + ....+ bnq)1/q. 


27
2.1.1.5. Bất đẳng thức Svacxơ
Cho 2n số thực a1, ..., an; b1, ..., bn, trong đó bi > 0, với i = 1, 2, ..., n, ta

2
2
a  a2  ...an 
a
a1
a2
.

 ...  n  1
b1
b2
bn
b1  b2  ...  bn
2

2

Chứng minh. Xét hàm lồi f(x) = x2, với x  R, áp dụng bất đẳng thức
Jensen dạng (2.6b) ta có

2

2
2
 m1 x1  ...  mn xn 
m x  ...  mn xn

  1 1
.
m1  ...  mn
 m1  ...  mn 

Hay
 m1 x1  ...  mn xn

 m1  ...  mn

Chọn mi = bi và xi =

2


  m1 x1 2  ...  mn x n 2 .


ai
, i = 1, 2, ..., n thì
bi

a1  a2  ...  an 2

b1  b2  ...  bn

Đó là điều phải chứng minh.

2

2

2

a
a
a
 1  2  ..  n .
b1
b2
bn



2.1.1.6. Bất đẳng thức Isena
Cho các số dương a, b và p, q thỏa mãn p, q >1, (1/p) + (1/q) = 1. Khi đó
ta có
ap
aq
 ab.
+
p
q


Chứng minh. Xét hàm lõm f(x) = lnx, x > 0. Theo bất đẳng thức Jensen đối
với hàm lõm, ta có
1
1
f (a p )  f (b q ) 
p
q


1
1 
f  a p  b q 
q 
p

1
1
1
1 
ln a p  ln b q  ln  a p  a q 
p
q
q 
p


28
1
1 
 ln ab  ln  a p  b q  .

q 
p

Do hàm f(x) = lnx đồng biến nên
a p bq

 ab .
p
q

Dấu “=” xảy ra khi và chỉ khi ap = bq. 
Chú ý: Ta có bất đẳng thức Isena tổng quát như sau
Cho các số dương a1, a2, ..., an và các số p1, p2, ..., pn thỏa mãn
p1, p2, ..., pn > 1,

1
1
1
= 1. Khi đó ta có

 ... 
p1 p2
pn

anpn
a1p1 a2p2

 ... 
 a1a2 ...an .
p1

p2
pn

2.1.2. Các bất đẳng thức đại số
2.1.2.1. Cho a, b, c > 0 chứng minh rằng
(b+c)a(c+a)b(a+b)c 

2
(a+b+c)a+b+c.
3

Giải:
Xét hàm lồi f(x) = - ln x; x > 0.
Theo bất đẳng thức Jensen với cách chọn

1 =

a
b
c
, 2 =
, 3 =
abc
abc
abc

x1 =

bc
ca

ab
, x2 =
, x3 =
,
2
2
2

ta có
a
b
c
b  c
c  a
 a  b
f
f
f



abc  2  abc  2  abc  2 
a
bc
b
ca
c
a  b

 f

.

.

.

abc 2
abc 2 
abc 2


×